Difference between revisions of "2007 iTest Problems/Problem 17"

m (Solution)
m
Line 2: Line 2:
  
 
If <math>x</math> and <math>y</math> are acute angles such that <math>x+y=\frac{\pi}{4}</math> and <math>\tan{y}=\frac{1}{6}</math>, find the value of <math>\tan{x}</math>.
 
If <math>x</math> and <math>y</math> are acute angles such that <math>x+y=\frac{\pi}{4}</math> and <math>\tan{y}=\frac{1}{6}</math>, find the value of <math>\tan{x}</math>.
 +
 +
<math>\text{(A) }\frac{37\sqrt{2}-18}{71}\qquad
 +
\text{(B) }\frac{35\sqrt{2}-6}{71}\qquad
 +
\text{(C) }\frac{35\sqrt{3}+12}{33}\qquad
 +
\text{(D) }\frac{37\sqrt{3}+24}{33}\qquad</math>
 +
 +
<math>\text{(E) }1\qquad
 +
\text{(F) }\frac{5}{7}\qquad
 +
\text{(G) }\frac{3}{7}\qquad
 +
\text{(H) }6\qquad</math>
 +
 +
<math>\text{(I) }\frac{1}{6}\qquad
 +
\text{(J) }\frac{1}{2}\qquad
 +
\text{(K) }\frac{6}{7}\qquad
 +
\text{(L) }\frac{4}{7}\qquad
 +
\text{(M) }\sqrt{3}\qquad</math>
 +
 +
<math>\text{(N) }\frac{\sqrt{3}}{3}\qquad
 +
\text{(O) }\frac{5}{6}\qquad
 +
\text{(P) }\frac{2}{3}\qquad
 +
\text{(Q) }\frac{1}{2007}\qquad</math>
  
 
== Solution ==
 
== Solution ==
Line 20: Line 41:
 
Rearranging and solving, we get
 
Rearranging and solving, we get
  
<math>\tan{x}=\boxed{\frac{5}{7}}</math>
+
<math>\tan{x}=\boxed{\textbf{(F) } \frac{5}{7}}</math>
  
 
==See Also==
 
==See Also==

Revision as of 16:57, 10 June 2018

Problem

If $x$ and $y$ are acute angles such that $x+y=\frac{\pi}{4}$ and $\tan{y}=\frac{1}{6}$, find the value of $\tan{x}$.

$\text{(A) }\frac{37\sqrt{2}-18}{71}\qquad \text{(B) }\frac{35\sqrt{2}-6}{71}\qquad \text{(C) }\frac{35\sqrt{3}+12}{33}\qquad \text{(D) }\frac{37\sqrt{3}+24}{33}\qquad$

$\text{(E) }1\qquad \text{(F) }\frac{5}{7}\qquad \text{(G) }\frac{3}{7}\qquad \text{(H) }6\qquad$

$\text{(I) }\frac{1}{6}\qquad \text{(J) }\frac{1}{2}\qquad \text{(K) }\frac{6}{7}\qquad \text{(L) }\frac{4}{7}\qquad \text{(M) }\sqrt{3}\qquad$

$\text{(N) }\frac{\sqrt{3}}{3}\qquad \text{(O) }\frac{5}{6}\qquad \text{(P) }\frac{2}{3}\qquad  \text{(Q) }\frac{1}{2007}\qquad$

Solution

From the second equation, we get that $y=\arctan\frac{1}{6}$. Plugging this into the first equation, we get:

$x+\arctan\frac{1}{6}=\frac{\pi}{4}$

Taking the tangent of both sides,

$\tan(x+\arctan\frac{1}{6})=\tan\frac{\pi}{4}=1$

From the tangent addition formula, we then get:

$\frac{{\tan{x}+\frac{1}{6}}}{{1-\frac{1}{6}\tan{x}}}=1$

$\tan{x}+\frac{1}{6}=1-\frac{1}{6}\tan{x}$.

Rearranging and solving, we get

$\tan{x}=\boxed{\textbf{(F) } \frac{5}{7}}$

See Also

2007 iTest (Problems, Answer Key)
Preceded by:
Problem 16
Followed by:
Problem 18
1 2 3 4 5 6 7 8 9 10 11 12 13 14 15 16 17 18 19 20 21 22 23 24 25 26 27 28 29 30 31 32 33 34 35 36 37 38 39 40 41 42 43 44 45 46 47 48 49 50 51 52 53 54 55 56 57 58 59 60 TB1 TB2 TB3 TB4